LSAT and Law School Admissions Forum

Get expert LSAT preparation and law school admissions advice from PowerScore Test Preparation.

 Adam Tyson
PowerScore Staff
  • PowerScore Staff
  • Posts: 5153
  • Joined: Apr 14, 2011
|
#66798
Answer A isn't saying that a comprehensive approach WILL solve the problem, whardy21. It's just saying that if the problem can be solved, it would require a comprehensive approach. The evidence for this is strong - the "current piecemeal system" cannot do the job. If we accept the statements in the stimulus, which we are supposed to do for this type of question, and if we presume that there are only two options (piecemeal and comprehensive, which seem to make a pretty good pair for logical opposition), then answer A is very strongly supported.

But in the end, your analysis is still correct. The Fact Test supports answer A, and it does not support any of the other answers. Whether you believe this means A must be true or not, it is for sure the answer that is most strongly supported, and so it must be the credited response.

Well done!
 andriana.caban
  • Posts: 142
  • Joined: Jun 23, 2017
|
#71097
Eric Ockert wrote:Hi co!

Careful. This isn't a Strengthen question, it is a Must Be True question. Here we are using the argument in the stimulus to support one of the answer choices, not the other way around.

You are right that the author concludes that comprehensive solutions cannot be found in the current piecemeal system. But if that is the case, the only solution to healthcare costs must be found outside of this system. And that is exactly what answer (A) is saying. So (A) is provable based on the statements in the stimulus, and therefore correct as a Must Be True answer.

Hope that clears it up a bit!

Hi!

How is (A) saying that the only solution to healthcare costs must be found outside of the system. It says, "under the conditions in which the current system operates" and follows with healthcare costs could be decreased (if any) only by a comprehensive approach. It seems to me that (A) is discussing plans inside our current piecemeal system.

To me, this seems like a most strongly supported question. My prephrase sounded something along the lines of "if we observe a coherent solution existing that reduces the health care costs, it's probably not going to be in our current system".

Answer (A) seems to hold a degree of uncertainty to me, which made me hesitate for a moment. Under the current system, the overall costs of healthcare can be reduced by a comprehensive approach did not follow 100 % from the stimulus. If we have a comprehensive approach in our current system, then the problems plaguing the current system can still deem it ineffective.

I only chose (A) through POE because all of the other answer choices were way out of bounds, but it kind of irks me that I still can't understand why (A) follows logically from the stimulus.

If anyone can help I would really appreciate it - thanks!!!!!
User avatar
 KelseyWoods
PowerScore Staff
  • PowerScore Staff
  • Posts: 1079
  • Joined: Jun 26, 2013
|
#71747
Hi Andriana!

A "comprehensive approach" is basically the opposite of the current "piecemeal system." They cannot coexist. I think the confusion comes over what the author is referring to by "system." The system is how we approach healthcare costs. Currently, it's a piecemeal system. The only way to lower healthcare costs is by switching to a comprehensive system. You can't have a comprehensive system WITHIN a piecemeal system. So we have to get rid of the old piecemeal system so we can have a comprehensive system in its place. When the author says "Under the conditions in which the current system operates" the author is referring to the larger healthcare conditions. So currently, under those conditions we have the piecemeal system. To lower healthcare costs under these conditions, we need to have a comprehensive system instead. So we're still in the confines of the healthcare conditions, but we're going OUTSIDE of the piecemeal system to lower costs with a completely different, comprehensive system.

Hope this helps!

Best,
Kelsey
 kalifaingold
  • Posts: 11
  • Joined: Jan 14, 2020
|
#73354
KelseyWoods wrote:Hi Andriana!

A "comprehensive approach" is basically the opposite of the current "piecemeal system." They cannot coexist. I think the confusion comes over what the author is referring to by "system." The system is how we approach healthcare costs. Currently, it's a piecemeal system. The only way to lower healthcare costs is by switching to a comprehensive system. You can't have a comprehensive system WITHIN a piecemeal system. So we have to get rid of the old piecemeal system so we can have a comprehensive system in its place. When the author says "Under the conditions in which the current system operates" the author is referring to the larger healthcare conditions. So currently, under those conditions we have the piecemeal system. To lower healthcare costs under these conditions, we need to have a comprehensive system instead. So we're still in the confines of the healthcare conditions, but we're going OUTSIDE of the piecemeal system to lower costs with a completely different, comprehensive system.

Hope this helps!

Best,
Kelsey
Hello, How am I supposed to know that a "comprehensive approach" is basically the opposite of the current "piecemeal system"? This was not obvious at all to me, as nowhere in the stimulus does it state that or even say the word comprehensive at all. To me, every answer seemed way off base! But I would have never chosen A since it doesn't say that a comprehensive approach would shrink health care costs.
 Robert Carroll
PowerScore Staff
  • PowerScore Staff
  • Posts: 1783
  • Joined: Dec 06, 2013
|
#73357
kalifa,

A "piecemeal" approach by definition involves handling only isolated parts. A comprehensive approach would be one that did not settle for handling only parts, but attempted to deal with the whole. The stimulus does not contain these definitions because they are part of the meanings of these terms and don't have to be defined.

I think you're saying that the stimulus doesn't say that a comprehensive approach would shrink health care costs. And that's true! Answer choice (A) doesn't say that either - note the necessary condition indicator "only". The answer is not saying that a comprehensive approach will work; instead it's saying that, if ANYTHING could work, it would have to be comprehensive. And that's true according to the stimulus because the first sentence tells us that a piecemeal approach will NEVER work. So, if anything ever does work, it must be a departure from the piecemeal approach - it must be comprehensive.

Robert Carroll
 intrepidlady97
  • Posts: 4
  • Joined: May 16, 2022
|
#95376
Administrator wrote: Fri Jan 21, 2011 12:00 am Complete Question Explanation

Must be True. The correct answer choice is (A)

Although technically the answer to this question does not have to be true, this is still a Must Be True question and is approached the same way. From the stimulus, we know that the current piecemeal system is not working, cannot provide any coherent solutions, and gives health care providers and insurers every incentive to shift health care costs upon patients or each other. So, the question to ask ourselves her is what conclusion results naturally from that information. If you were to prephrase an answer here, it would probably have something to do with the fact that a more complete plan in necessary.

Answer Choice (A): This is the correct answer choice. The stimulus tells us outright that solutions to the problem cannot be found in the current piecemeal system. Therefore, it follows that the only possible means of reducing health care costs is through a more comprehensive approach.

Answer Choice (B): There is nothing in the stimulus addressing the income of health care professionals, and thus there is nothing in the stimulus that we can look at to support this answer choice.

Answer Choice (C): Again, since there is nothing in the stimulus regarding the expansion of health care funds relative to health care costs, we can find no support for this answer choice.

Answer Choice (D): The stimulus does not address any technological issues or any costs attributed to technology. Once again, there is nothing in the stimulus to support this answer choice.

Answer Choice (E): One last time, since there is nothing in the stimulus addressing anything close to the issue in this answer choice, there is nothing to support this answer choice.
I am wondering why you have classified this question as a Must be True question? It reads as a Most Strongly Supported question to me: "The argument provides the most support for which of the following"
 Robert Carroll
PowerScore Staff
  • PowerScore Staff
  • Posts: 1783
  • Joined: Dec 06, 2013
|
#95420
intrepidlady97,

Most Strongly Supported questions are a subtype of Must Be True. We classify all Most Strongly Supported questions as Must Be True. The only difference between these subtypes is that the correct answer for a strict Must Be True question must be 100% proven by the stimulus, whereas the correct answer for a Most Strongly Supported question might not be 100% proven, although it will have to be an extremely good, almost perfect, inference from the stimulus.

Robert Carroll
User avatar
 TSimmons
  • Posts: 7
  • Joined: May 06, 2022
|
#96307
Robert Carroll wrote: Fri Jan 17, 2020 5:08 pm kalifa,

A "piecemeal" approach by definition involves handling only isolated parts. A comprehensive approach would be one that did not settle for handling only parts, but attempted to deal with the whole. The stimulus does not contain these definitions because they are part of the meanings of these terms and don't have to be defined.

I think you're saying that the stimulus doesn't say that a comprehensive approach would shrink health care costs. And that's true! Answer choice (A) doesn't say that either - note the necessary condition indicator "only". The answer is not saying that a comprehensive approach will work; instead it's saying that, if ANYTHING could work, it would have to be comprehensive. And that's true according to the stimulus because the first sentence tells us that a piecemeal approach will NEVER work. So, if anything ever does work, it must be a departure from the piecemeal approach - it must be comprehensive.

Robert Carroll
Hi all,

The only reservation about immediately selecting (A) was whether 'comprehensive' was the logical/complete/formal negation of 'piecemeal.' In normal conversation, piecemeal and comprehensive would certainly be opposite, but I was applying lesson one's understanding of Hot and Cold not being formal negations, whereas Not Hot and Not Cold would be the case. It's a little silly bc 'not piecemeal' is ostensibly synonymous w 'comprehensive,' but it did eat up some time exploring alternatives between piecemeal and comprehensive--perhaps the 'lukewarm' equivalent between hot and cold. Maybe I'm getting too in the weeds. Thanks!
 Rachael Wilkenfeld
PowerScore Staff
  • PowerScore Staff
  • Posts: 1358
  • Joined: Dec 15, 2011
|
#96415
Hi TSimmons,

This is a great question. The key here is in the definitions and meanings of the two different words. Piecemeal means doing something one part at a time. The opposite of doing something one part at a time is not doing it one part at a time. If it's not one part at a time, that's the same as saying doing all the things together. You either do all the things (comprehensive) or not all of the things (piecemeal). Those are logical opposites of each other in a way that hot and cold are not.

Hope that helps!

Get the most out of your LSAT Prep Plus subscription.

Analyze and track your performance with our Testing and Analytics Package.